Choose the correct answer below.
A. The statement is true because the definition of the instantaneous rate of change for a function f at x is the same as the definition of the derivative of a function f at x
B. The statement is false because the derivative is the average rate of change of x with respect to y=f(x).
C. The statement is false because the derivative is the average rate of change of y=f(x)with respect to x
D. The statement is false because the derivative is the instantaneous rate of change of x with respect to y=f(x)

Choose The Correct Answer Below.A. The Statement Is True Because The Definition Of The Instantaneous

Answers

Answer 1

The correct answer to this question is A.


Related Questions

How to simplify this expression: 5-2(x+5)+2​

Answers

Answer:

........

Step-by-step explanation:

...

....

.....

......

.......

how to simplify (3x)³+2x³​

Answers

Answer: 5x³

Step-by-step explanation:

WILL GIVE BRAINLIEST!! PLEASE
A car travels 12 km North and then turns left to travel 5 km West before coming to a stop. What is the car’s displacement?

Answers

The answer will be 13 km that is displacement of the car.

What is displacement?

When a body shifts from one position to another, displacement is the smallest (straight line) distance between the starting position and the ending position of the body, which is symbolized by an arrow pointing from the starting position to the ending position. Displacement is a vector quantity that describes "how far out of place an object is"; it represents the overall change in the position of the object.

A car travels 12 km north and turns left to travel 5km.

So the final position of car from the initial position is :

So the placement of the car is √(12²+5²) = √169 =13km

Hence the Displacement of the car is 13 km

Learn more about Displacement, by the following link

https://brainly.com/question/321442

#SPJ1

Question 3.13R:
Which of the following are equivalent exponential function(s) to f(x)=(4/7)^−x

Answers

The equivalent function of the exponential function is (a) f(x) = (7/4)ˣ

How to determine the equivalent function?

From the question, we have the following function that can be used in our computation:

f(x) = (4/7)^-x

This function can be properly expressed as

f(x) = (4/7)⁻ˣ

Solving further, we need to apply the exponent law of indices

This is represented as

a⁻ˣ = 1/aˣ

When the above rule is applied, we have

f(x) = 1/(4/7)ˣ

Evaluate the quotient

f(x) = (7/4)ˣ

Hence, the equivalent is (a) f(x) = (7/4)ˣ

Read more about exponential function at

https://brainly.com/question/11464095

#SPJ1

An amount of $ 48,000$ is borrowed lord 9 years at $6 \% 6 interest, compounded annually. If the loan is paid in full at the end of that period, thaw Mach must be paid back?
Use the calculator provided and round your answer to the rarest dollar.

Answers

By using the formula for compound interest, it can be calculated that

He must pay back $81095 after 9 years

What is Compound interest?

If the interest on a certain principal at a certain rate over a certain period of time increases exponentially, rather than linearly, then the interest earned is called compound interest

If p is the principal, r is the rate and t is the time, then the amount is calculated by the formula

[tex]A = p(1 + \frac{r}{100})^n[/tex]

The difference between the amount and the principal gives the compound interest.

Principal = $48000

Rate = 6%

Time = 9 years

Amount = [tex]48000(1+ \frac{6}{100})^9\\[/tex]

             = $81095

He must pay back $81095 after 9 years

To learn more about compound interest, refer to the link-

https://brainly.com/question/24274034

#SPJ4

The expression −290+18y represents a submarine that began at a depth of 290 feet below sea level and ascended at a rate of 18 feet per minute. What was the depth of the submarine after 6 ​minutes?

Answers

Answer: would be -120 + 78, or -42 feet.

Step-by-step explanation:

How to find standard deviation using the range of the sample set?

Answers

The range of the data divided by four gives the standard deviation, which is approximately.

How do you find the sample standard deviation from the range?

The range of the data divided by four gives the standard deviation, which is approximately. Simply put, that's it. To determine the range, find the highest value—the maximum—and deduct the lowest—the minimum. The range should then be divided by 4.

The range rule states that a sample's standard deviation is roughly equal to one-fourth of the data range.

In order to calculate the standard deviation, we must first determine the mean, take this mean away from each data point, square the differences, add them, divide by one less than the number of data points, and (at the very end) take the square root. The range rule, however, only needs one division and one subtraction.

To learn more about standard deviation refer to:

https://brainly.com/question/475676

#SPJ1

A rectangle has a width represented by the expression-2x+20.The length of the rectangle is 6 more than twice the width.which expression represents the area of the rectangle?

Answers

Answer:

8(x^2)+172x+920

Step-by-step explanation:

width=2x+20length=2(width)+6

2(2x+20)+6

=4x+46

area=lenght ×width

(4x+46)×(2x+20)

=4x(2x+20)+46(2x+20)

=8(x^2)+80x+92x+920

=8(x^2)+172x+920

Compute the amount in an account after 4 years if $4550 is invested at an annual interest rate of 3.5% compounded semiannually. Round to two decimals.

Answers

Answer:

$5,227.41

Step-by-step explanation:

The compound interest formula is

[tex]A=P(1+\frac{r}{n}) ^{nt}[/tex]

A is the final amount

P is the initial amount

r is the interest rate as a decimal

n is the # of times the interest is applied per time period

t is the # of time periods.

We are given

Note: semi annually is twice per year.

[tex]P=4550[/tex]

[tex]r=0.035[/tex]

[tex]n=2[/tex]

[tex]t=4[/tex]

Lets solve for A.

[tex]A=4550(1+\frac{0.035}{2} )^{2*4}[/tex]

[tex]A=4550(1.0175)^{8}[/tex]

[tex]A=4550(1.14888178)[/tex]

[tex]A=5227.41211244[/tex]

Round to two decimals.

$5,227.41

Which phrase represents this expression? (6242) x 3 O 3 times the difference of 42 and 62 O the difference of the product of 3 times 42 and 62 O the difference of 62 and the product of 3 times 42 O 3 times the difference of 62 and 42.

Answers

By using the concept of mathematical expression, it is obtained that

(62−42)×3 is represented by  3 times the difference between 62 and 42

First option is correct

What is mathematical expression?

A mathematical expression consist of numbers connected with addition, subtraction, multiplication and division.

Here the given  mathematical expression is (62−42)×3

Here, the signs shown are subtraction and multiplication. So the concept of difference and times will be used here

(62−42)×3  means whole of (62 - 42) is multiplied with 3

So, this is 3 times the difference between 62 and 42

First option is correct

To learn more about mathematical expression, refer to the link-

https://brainly.com/question/723406

#SPJ9

Correct Question

Which phrase represents this expression?

(62−42)×3

a) 3 times the difference of 62 and 42

b) 3 times the difference of 42 and 62

c) The difference of 62 and the product of 3 times 42

d) The difference of the product of 3 times 42 and 62

A house has increased in value by 15% since it was purchased. If the current value is 299,000, what was the value when it was purchased?

Answers

The value of the house when it was purchased is 260,000 units

How to find the value the house was purchased?

The house has increased in value by 15 percent since it was purchased. The current value is 299,000.

The value of the house when it was purchased can be calculated as follows:

let the amount purchased = x

Hence,

15% of x + x  = 299000

0.15x + x  = 299000

1.15x = 299000

divide both sides by 1.15

Hence,

x = 299000 / 1.15

x = 260000

Therefore, the value of the purchase is 260,000 units

learn more on percentage here: https://brainly.com/question/18881739

#SPJ1

Convert .51 kg to milligrams using unit fractions

Answers

The conversion of .51 kg to milligrams using unit fractions is given by 510,000 mg

How to convert from kilograms to milligrams?

Units can be converted from one form to another.

Kilograms written as kg is a unit of massMilligrams written as mg is also a unit of mass.

Convert kilogram to milligram:

1000g = 1 kg

1 kg = 1,000,000 mg

That is,

A kilogram is equivalent to one million milligrams.

.51 kg to milligrams

= .51 × 1,000,000

= 510,000 mg

Hence, the conversion of .51 kg to milligrams is 510,000 mg.

Read more on kilograms to milligrams:

https://brainly.com/question/28635375

#SPJ1

Differentiate the function with respect to x. Shot steps

Answers

The differentiated form of the function with respect to x will be e³ˣ³.9x².

What is a function?

It is defined as a special type of relationship, and they have a predefined domain and range according to the function every value in the domain is related to exactly one value in the range. The exponential function is that rapidly increases and the value of the exponential function is always positive. It denotes with exponent y = a^x where a is a constant and a>1.

It is given that the function is,

y=e³ˣ³

Differentiate the function with respect to x.

[tex]=\frac{dy}{dx} \\\\ =\frac{d}{dx}\left(\left(e^3\right)^{x^3}\right) \\\\ =e^{3x^3}\frac{d}{dx}\left(3x^3\right) \\\\ =e^{3x^3}\cdot \:9x^2[/tex]

Thus, the differentiated form of the function with respect to x will be e³ˣ³.9x².

Learn more about the function here:

brainly.com/question/5245372

#SPJ1

Yesterday, you entered into a futures contract to sell 62,500 at $1.50 per . Your initial performance
bond is $1,500 and your maintenance level is $500. At what settle price will you get a demand for additional
funds to be posted?

Answers

The settle price at which you will get a demand for additional funds to be posted is $1.4840 per €.

What is contract?

An agreement between parties that results in legal duties for both parties is known as a contract. Mutual consent, demonstrated by a valid offer and acceptance, sufficient consideration, capability, and legality are the fundamental components needed for the agreement to be a legally enforceable contract.

Provided data from the question;

Futures contract to buy in euros = €62,500

Future contract to buy in dollar = $93,750 ( €62,500 × $1.50)

Initial performance of bond = $1,500

Maintenance level = $500

To determine the settle price at which you get a demand for additional funds to be posted =

Future contract to buy in dollar - (initial bond performance - maintenance level) ÷ future contract to in euros

$93,750 - ($1,500 - $500)

= $93,750 - $1,000

$92,750 ÷ €62,500

= $1.484

To learn more about the fund deposit from the given link

https://brainly.com/question/28704231

#SPJ1

Five subtracted from seven times a number is 86

Answers

Answer:

7x-5=86

Step-by-step explanation:

Use the arc length formula to find the length of the graph of the function y = x ^ (3/2) + 4 from x = 2 to x = 9 (Express numbers in exact form. Use symbolic notation and fractions where needed.)

Answers

Answer:

[tex]\frac{1}{27}((13)^{\frac{3}{2}}-(6)^ {\frac{3}{2}})[/tex]

Step-by-step explanation:

PLEASE HELP ASAP!!! Solve the problem.
Triangle KAP = triangle __ by rule __

Answers

Step-by-step explanation:

Statements                                                Reasons

1. AK - angle bisector                                 1. Given

2. FP perpendicular AK                             2. Given

3. <FAK congr <PAK                                  3. Definition angle bisector

4. Seg AK congr seg AK                           4. Congr of seg's is reflexive

5. <AKF & <AKP are rt. angles                  5. Def perpendicular lines

6. <AKF is congr <AKP                              6. Thrm: Right angles are congr.

7. Triangle KAF congr triangle KAP         7. ASA

8. A_F = A_P                                                    8. Corresp parts of congr tr are

                                                                           congr

A toy rocket is shot vertically into the air from a launching pad 7 feet above the ground with an initial velocity of 72 feet per second. The height h, in feet, of the rocket above the ground at t seconds after launch is given by the function h(t)=-16 t²+72 t+7. How long will it take the rocket to reach its maximum height? What is the maximum height?
The rocket reaches its maximum height at ______ second(s) after launch.
(Simplify your answer.)
The maximum height reached by the object is ______feet.
(Simplify your answer.)

Answers

The time taken for the rocket to reach its maximum height as required in the task content is; 2.25 seconds.

The maximum height reached by the toy rocket is; 88 feet.

What is the maximum height reached by the rocket?

It follows from the task content that the maximum height reached by the rocket is to be determined.

The given function is; h (t) = -16 t² + 72t + 7.

Since it follows from evaluation that; at maximum height, the rate of change of height to time is equal to 0.

Therefore; at maximum height;

h'(t) = 0

h'(t) = -32t + 72 = 0

-32t = -72

t = -72/-32

t = 2.25 seconds.

Therefore, the time taken for the rocket to reach its maximum height is; 2.25 seconds.

Also, the maximum height reached by the rocket is at; h (2.25);

h(2.25) = -16(2.25)² + 72 (2.25) + 7

= -81 + 162 + 7

= 88.

The maximum height reached is; 88 feet.

Read more on maximum height;

https://brainly.com/question/82347

#SPJ1

Question 36 of 50
When graphing systems of inequalities, the possible answers are all the points that are in the overlapped, shaded region of the two lines.

True

False

Answers

The solutions to a system of linear inequalities are located in the region where all the shaded regions of the inequalities overlap. the region of overlap called the intersection.        

What is inequality?

It is defined as the expression in mathematics in which both sides are not equal they have mathematical signs either less than or greater than known as inequality.

For the inequality, we have to apply the arithmetic operation in which we do the multiplication of x and apply the inequality for the given data.

Thus, the solutions to a system of linear inequalities are located in the region where all the shaded regions of the inequalities overlap. the region of overlap called the intersection.        

Learn more about inequality here:

brainly.com/question/19491153

#SPJ1

The table shows the number of each type of cookie a
bakery has left at the end of the day. The baker wants to
make the greatest number of cookie boxes to sell, using
chocolate chip and sugar cookies together. If all of the
chocolate chip and sugar cookies are distributed evenly
among the boxes and the baker charges $5 per box, how
much money will the bakery bring in if they sell all of the
boxes?

Answers

$65

the most boxes that can be mad is 13 boxes (so 5 in each).


explanation:

cc+sc= 65 cookies

65/5=13 (so 13 boxes)

13 boxes times $5 = $65

Determine the length of the line segment shown. line segment from negative 5 comma 5 to 3 comma negative 1

6 units
8 units
10 units
36 units

Answers

The distance of the line segment from ( -5,5 ) to ( 3,-1 ) is 10 units

What is the distance of a line between 2 points?

The distance of a line between 2 points is always positive and given by the formula

[tex]D = \sqrt{(x_{2}-x_{1})^{2} + (y_{2}-y_{1})^{2} }[/tex]

Given data ,

Let the two points be A and B

A ( x₁ , y₁ ) = A ( -5 , 5 )

B ( x₂ , y₂ ) = B ( 3 , -1 )

So , the distance between 2 points is given by the formula

[tex]D = \sqrt{(x_{2}-x_{1})^{2} + (y_{2}-y_{1})^{2} }[/tex]

Substituting the values of x₁ , y₁ , x₂ , y₂ in the formula , we get

[tex]D = \sqrt{(-5-3)^{2} + (5-(-1))^{2} }[/tex]

[tex]D = \sqrt{(8)^{2} + (6)^{2} }[/tex]

[tex]D=\sqrt{(64+36)}[/tex]

[tex]D = \sqrt{100}[/tex]

[tex]D= 10 units[/tex]

Hence , the distance of the line is 10 units

To learn more about distance of line between 2 points click :

https://brainly.com/question/18234495

#SPJ1

Graphing and Solving Inequalities
m-1<-3
a
-4y <-20
5+b>1

-18
There is a graph underneath each one with lines and arrows.

Answers

The solution to inequalities are as follows.

What is inequality equation?

Inequalities are the mathematical expressions during which either side don't seem to be equal. In difference, in contrast to in equations, we tend to compare 2 values. The equal sign up between is replaced by but (or but or equal to), larger than (or larger than or equal to), or not capable sign.

Main body:

A) m-1 <-3

   to solve this we need to take link terms to one side

  = m< -3+1

  = m < -2

B) -4y <-20

  cancelling -ve sign on both sides,

= 4y <20

= y <5

C) 5+b>1

take link terms to one side

b > -4

Hence the graph is as given

to know more about inequality equation click on the link below

https://brainly.com/question/11613554

#SPJ1

A car leaves Rockford, Illinois, at the rate of 70 mph, bound for Madison, Wisconsin. At the same time, a truck leaves Madison at the rate of 50 mph, bound for Rockford. If the cities are 72 miles apart, how long will it take for the car and the truck to meet?

Answers

Answer:

36 minutes

Step-by-step explanation:

The total distance covered by the two vehicles per hour: 70 + 50 = 120 miles.

Time is distance over velocity.

[tex]t=\frac{72}{120}[/tex]  hours

Which simplifies to

[tex]\frac{3}{5}[/tex]  hours.

Convert hours to minutes.

There is 60 minutes in 1 hour.

[tex]\frac{1}{60}=\frac{\frac{3}{5} }{x}[/tex]

[tex]x=36[/tex]

Solving -8 + x > -9 for x is: x > -1.

True

False

Answers

Step-by-step explanation:

Solving -8 + x > -9 for x is: x > -1.

-8 + x > -9

add 8 to both sides:

-8 + x + 8 > -9 + 8

x > -1

True

What is the F ratio in ANOVA?

Answers

Answer:

The F value is used in analysis of variance (ANOVA). It is calculated by dividing two mean squares. This calculation determines the ratio of explained variance to unexplained variance. The F distribution is a theoretical distribution.

Step-by-step explanation:

May I have Brainliest please? My next rank will be the highest one: A GENIUS! Please help me on this journey to become top of the ranks! I only need 15 more brainliest to become a genius! I would really appreciate it, and it would make my day! Thank you so much, and have a wonderful rest of your day!

The F ratio in ANOVA is the difference between two mean square values. If

In ANOVA, what is the F ratio?

Analysis of variance is a collection of statistical models and the estimation procedures that go with them that are used to analyze the differences between means. Ronald Fisher, a statistician, invented ANOVA. The ANOVA test compares more than two groups at the same time to see if there is a relationship between them.

If the null hypothesis is correct, you should expect F to be close to 1.0 most of the time. A high F ratio indicates that the variation among group means is greater than would be expected by chance.

Learn more about ANOVA on;

https://brainly.com/question/25800044

#SPJ4

How do you calculate p-value from z-score?

Answers

The procedure to calculate the p-value from z-score depends on one of two choices: one-tailed (which may be Left-tailed or Right-tailed) or two-tailed tests.

How to calculate p-value from z-score?

A z-table, otherwise termed the standard normal table, provides the area under the curve to the left of a z-score.

The technique for using z-tables to find p-values depends on the test being carried out; which may be; one- or two-tailed test.

For one-tailed tests, the directionality is also a factor.

It therefore follows that;

For a Left-tailed z-test: the p-value = Φ(Zscore)

For a Right-tailed z-test: the p-value = 1 - Φ(z-score).

For a Two-tailed z-test: the p-value = 2 * Φ(−|z-score|).

The procedure described above represents how to calculate p-value from z-score.

Read more on p-value;

https://brainly.com/question/13873630

#SPJ1

The following formulas can be used to determine the p-value from its corresponding z-score

Left-tailed test: Φ(z-score)Right-tailed test: 1 - Φ(z-score)Two-tailed test: 2Φ(−|z-score|)

How to calculate p-value from its z-score?

From the question, the parameters are given as

p-valuez-score

Also, from the question, we understand that:

The p value depends on the z-score for its own value

There are three types of test for z-score; these test are:

Right tailed testLeft tailed testTwo tailed test

The p-value would assume different value for a z-score when the type of test is different

This means that the p value for z = 0.2 at right tail test is different at a left tailed test.

This means that there are three formulas for calculating the p-value from the z-score, and these formulas are

Left-tailed test: Φ(z-score)Right-tailed test: 1 - Φ(z-score)Two-tailed test: 2Φ(−|z-score|)

Read more about p-value at

https://brainly.com/question/4621112

#SPJ4

x^2+8x+y^2+8y-112=0 center and radius

Answers

We can rewrite the equation of the circle as:

(x + 4)^2 + (y + 4)^2 = 12^2

So the center is (-4, -4) and the radius is 12 units.

How to find the center and radius of the circle?

For a circle of radius R and center (a, b) the equation is:

(x - a)^2 + (y - b)^2 = R^2

Here we have the equation:

x^2 + 8x + y^2 + 8y - 112 = 0

We can complete squares to get:

(x^2 + 8x) + (y^2 + 8y) - 112 = 0

(x^2 + 2*4*x + 4^2) +  (y^2 + 8y + 4^2) - 4^2 - 4^2 - 112 = 0

(x + 4)^2 + (y + 4)^2 = 112 + 16 +  16 = 144

(x + 4)^2 + (y + 4)^2 = 12^2

So we have a circle of radius of 12 units, with a center at (-4 , -4)

Learn more about circles:

https://brainly.com/question/1559324

#SPJ1

Simplify the expression. 11z +9-7=

Answers

Answer: 11z+2

Step-by-step explanation:

Raina needs to earn at least $2200 each month to cover her living expenses. Her income comes from a 4% commission that she earns on everything she sells. What is the minimum amount she needs to sell in order to earn $2200?

Answers

The person needs to sell a minimum amount of $550000 to earn $22000.

What is the percentage?

The percentage is a ratio that can be expressed as a fraction of 100.

Given that, the income of the person is 4% of the total selling value.

Let the person sells $x to earn $22000, therefore,

4% of x is $22000

Or, 4x/100 = 22000

     4x = 2200000

       x = 550000

Hence, the person needs to sell a minimum amount of $550000 to earn $22000.

Learn more on percentages here:

https://brainly.com/question/14979505

#SPJ1

What is the first term of the quotient of the following division problem?
(x³ - 1) = (x + 2)
O -0.5
O 1
O x²
Ox^4

Answers

The solution to the algebraic problem is [tex]x^{2}[/tex]

What is an Algebraic Equation?

An algebraic equation is a mathematical statement that sets two expressions equal to each other. An algebraic equation is typically made up of a variable, coefficients, and constants.

Solution:

([tex]x^{3}[/tex] – 1) = (x+2)

On dividing ([tex]x^{3} - 1[/tex]) by (x+2)

The first term of the quotient will be x^2 by following the Long Division Method of Algebraic Division.

To learn more about Algebraic Equation from the given link

https://brainly.com/question/4541471

#SPJ9

Other Questions
a month with 31 days has the same number of mondays and wednesdays.how many of the seven days of the week could be the first day of this month? Which interaction between magnets demonstrates the greatest change in kinetic energy? After which checkpoint is the cell first committed to continue the cell cycle through m?. 1.31 (a) possible coverage error: only employees in a specific division of the company were sampled. (b) possible nonresponse error: no attempt is made to contact nonrespondents to urge them to complete the evaluation of job satisfaction. (c) possible sampling error: the sample statistics obtained from the sample will not be equal to the parameters of interest in the population. (d) possible measurement error: ambiguous wording in questions asked on the questionnaire. why is hamlet considered by some to be a problem play? select all that apply. it contains sinful behavior in most of the scenes. it contains somber and humorous scenes. it contains a character who suffers needlessly. it contains morally ambiguous characters. lAshley's bill for dinner at a restaurant was $100. She left a 15% tip. What was the amountof the tip? the ionization energy of o2 is 1205 kj/mol. what is the maximum wavelength of light capale of causing the ionization of o2? Were was manufacturing done prior to factory being built Part A3A model car is for sale online. Beginning on day one, the owner discounts the price by 5% each day until it sells. On the third day thIf the original price of the model car was $40, how much did the car sell for to the nearest cent? True or FalseWrite T for True and F for False, Correct any false statements to make them true.1. Georgia's Indian tribes sided with the Americans during the Revolutionary War.2. Settlers did not fear any Indian attacks on the frontier.3. The Yazoo territory came under federal government control in 1802.4. William McIntosh was the first cousin of Governor Troup.5. The Treaty of Indian Springs ceded the remaining Creek lands to Georgia.6. William McIntosh was murdered by angry federal troops.7. By 1827, the Creek tribe had left Georgia.16 BR a client begins to experience drainage of small amounts of bright red blood from the tracheostomy tube 24 hours after a supraglottic laryngectomy. which is the best nursing action? salma has set up a lemonade stand outside her house and sells small cups and large cups of lemonade. each small cup holds 12 ounces of lemonade and each large cup hold 16 ounces of lemonade. salma sold 60 cups of lemonade totaling 920 ounces. determine the number of small cups sold and the number of large cups sold simplify (8.1)(8.1) (8.1)6 (8.1)7 (8.1)8 (8.1)9 Monopoly profit will _____ when a monopolist goes from single-price monopoly to perfect price discrimination. please help me with this ERISA mandates that all employees must have the opportunity to be vested in their pension so thatA: the plan will continue to earn money regardless of the state of the stock marketB: companies can offer pension plans to domestic partners as well as married couplesC: the plan will stay with the emplyoyee regardless of whether the employee stays with the companyD: the plan will be guaranteed by the Pension Benefit Guaranty Corporation what are the dietary carbohydrate recommendations for athletes? how do these differ from the general population? if income increases from $20,000 to $30,000 and $1,000 of the new income is saved (but a total of $6,000 of the $30,000 is saved), then the apc is . Write the equation of the line that passes through the points (1,-3) and (3,3) if the time to consume is longer than the time to produce, then multiple concurrent consumers can be used to keep the producer busy at all times. determine the number of concurrent consumers and the number of buffers needed when c